PSAT Reading Question 394: Answer and Explanation

Question: 394

Which choice provides the best evidence for the answer to the previous question?

  • A. Lines 34–36 ("On the . . . fewer")
  • B. Lines 41–44 ("Several . . . Higgs boson")
  • C. Lines 44–45 ("Of course, . . . Standard")
  • D. Lines 55–58 ("And while . . . particle"')

Correct Answer: C

Explanation:

(C) These lines most directly support the notion that one needs both mathematical and observational consistency to have a sound theory. Choice (A) focuses on the difficulty of finding the Higgs boson. Choice (B) focuses on different theories about the Higgs. Choice (D) focuses on the hope that the Higgs boson will be discovered in the near future.

Test Information

  • Use your browser's back button to return to your test results.
  • Do more Reading tests.

More Tests

    All content of site and practice tests © 2022 Jack.
    Quick View

    PSAT Practice Tests

    More Information